Problem Set #1
by Tommy2000, Nov 29, 2016, 5:29 AM
Hmm, I guess I'll record some problems I recently did/had explained to me word for word '-' I'll post sols later if I have time...
1) Show that if
is prime for
, then
are primitive roots.
Solution
2) Prove that for
,
Solution
3) USAMO 2005/3
This problem isn't that fun...so I'm too lazy to post a solution.
4) WOOT 11/25 PoTD
Copied from Jeffery because too lazy to type
5) USACO Dec 15 Plat #1-3
Max Flow
High Card Low Card
Counting Haybales
6) 2016 PUMaC Finals #1-3
2016 PUMaC Finals 3
7) Find all
satisfying
and
.
Solution
8) (2016 Turkey TST #5) Find all functions
such that
and
.
Solution
It's also worth noting this nice result in fractals' solution. It was something I tried to prove, but ultimately could not.
1) Show that if



Solution
Suppose
is not a primitive root. Then
. By QR Law,
. However,
, so contradiction.
Suppose
is not a primitive root. Then
. By QR Law,
. However,
, so contradiction.




Suppose




2) Prove that for

![\[ \sum_{m = 1} ^ p \left( \sum_{n = 1} ^ h \left( \frac{m + n}{p} \right) \right) ^ 2 = h (p - h) \]](http://latex.artofproblemsolving.com/4/f/e/4fe29ff7873efd8744184b1018d3c742090f6c81.png)
Let
. We can write the sum as
In the first part, all are equal to
unless
, in which case it is
. Thus, the first part contributes
to the sum.
Claim:
for all
.
Proof
Since
, we can plug this in to obtain a sum of
as desired.

![\[ h \sum_{i = 1} ^ p \left( \frac{i}{p} \right) ^ 2 + 2 \sum_{k = 1} ^ h (h - k) S_k \]](http://latex.artofproblemsolving.com/c/4/4/c44f103ebd911d969aa45349cead8d1d1a482846.png)




Claim:


Proof
Note that
It suffices to find when
for some
. We can divide through by
(it's nonzero) to obtain an equation of the form
where
is nonzero. It's clear that the
in this are bijective with the
in the previous equation. We can write
as
and
for some
.
cannot be
or else
. Since
and
yield the same pair, we see there are
values of
which yield
. On the other hand, there are
values of
which yield a nonzero value. Hence, there are
values of
which yield
. Thus,
as desired.
![\[ S_k = \sum_{i = 1} ^ p \left( \frac{i ^ 2 + ik}{p} \right) = \sum_{i = 1} ^ p \left( \frac{(i + k/2) ^ 2 - (k / 2) ^ 2}{p} \right) \]](http://latex.artofproblemsolving.com/9/4/9/9492612a13b46fe382d0fe4ed1e11a83eb7f01d3.png)

























Since


3) USAMO 2005/3
This problem isn't that fun...so I'm too lazy to post a solution.
4) WOOT 11/25 PoTD
Copied from Jeffery because too lazy to type
Let
. Note that
and
so
is cyclic, so
lies on the perpendicular bisector of
. Since we want to show that the locus of
is a line perpendicular to
, which is parallel to
, we want
, which occurs when
. But, this is true as
. Thus, the result follows.












5) USACO Dec 15 Plat #1-3
Max Flow
For each flow, add 1 to each endpoint, subtract 1 from LCA and parent of LCA. Then to compute number of flows which go through a node, do a DFS.
High Card Low Card
Do it as if whole game were high card from beginning and whole were low card from end. Then this works by greedy.
Counting Haybales
Direct application of Segment Tree.
6) 2016 PUMaC Finals #1-3
2016 PUMaC Finals 3
We proceed by complex bashing. Let
be the unit circle. Then we have,

Now, since
is inside the quadrilateral, we have
iff
Hence, the two angles are equal.


Now, since



7) Find all



Solution
We claim
is odd. Suppose
, then
and
. Clearly these cannot be equal in magnitude unless
, in which case
is already odd.
is either
or
. However, if
,
and this contradicts
. Hence,
. Observe that
. Now, suppose
such that
. Then
. However,
is negative. Since
is odd and
, we conclude
, a contradiction. Thus, for
,
.
In particular,
. We claim
is increasing. Suppose
and
. Then we know that
. Since we know [that
is bounded between floor and ceil, this is a contradiction for sufficiently large
. Now notice that numbers of the form
for integers
are dense in reals, so we're done.






















![$f(c) \in [0, 1]$](http://latex.artofproblemsolving.com/d/6/9/d69c715e5e86ee1cd3f5364e1fd7ea53d1237f83.png)
In particular,
![$f(k) \in [\lfloor k \rfloor, \lceil k \rceil]$](http://latex.artofproblemsolving.com/0/3/2/032622cb8582feb4cfa8533d59ac1b49bf94b55e.png)








8) (2016 Turkey TST #5) Find all functions



Solution
Let (1) denote the first equation and (2) denote the second equation. Notice
by plugging
in (2). Furthermore, it's clear
for all
and
by (1).
[We claim that the answer is
for
. Since
is completely multiplicative, it suffices to find the values for
for prime
.
We first show that for each
,
is an odd power of
. First suppose
is not a power of
, then there is a prime
such that
. It's clear that there exists
such that
. Then note that
However, we know
, so this is a contradiction. Hence,
is a power of
. Now, observe that
for
, so it must be an odd power.
It remains to show that all of the primes have the same exponent. Take two arbitrary primes
with
and
. WLOG
. Observe that
. By (2) on
, we get
Applying the euclidean algorithm, we see
However,
is arbitrary, so this is a contradiction for sufficiently large
. Hence, the answer is indeed
for
.





[We claim that the answer is





We first show that for each









![\[ 1 + pm = qn \mid f(1) + f(pm) = 1 + f(pm) \]](http://latex.artofproblemsolving.com/5/8/9/589fe59975b6800a310e14f0e31623f7282fb017.png)





It remains to show that all of the primes have the same exponent. Take two arbitrary primes






![\[ p + q ^ x \mid p ^ a + \left( q ^ x \right) ^ b \]](http://latex.artofproblemsolving.com/f/3/f/f3feb186f8518dcdcb19ea05c264ac05aef11d31.png)
![\[ p + q ^ x \mid p ^ {a - b} - 1 \]](http://latex.artofproblemsolving.com/1/6/5/165b8622fabc0a1eb15f5f096ae407c306365631.png)




It's also worth noting this nice result in fractals' solution. It was something I tried to prove, but ultimately could not.
This post has been edited 15 times. Last edited by Tommy2000, Dec 7, 2016, 2:51 AM
Reason: Subject name
Reason: Subject name
Reason: RE